Probability proof. Check my answer?












0












$begingroup$


Let $A$ and $B$ be events defined on a sample space $Omega$. Prove that



$$textbf{P}(A cap B) geq 1 - textbf{P}(A^{c})- textbf{P}(B^{c}) $$



Answer:



begin{align*}
textbf{P}(Acap B) & = textbf{P}(A)textbf{P}(B) \
& = (1 - textbf{P}(A^{c}))(1 - textbf{P}(B^{c})) \
& = 1 - textbf{P}(A^{c}) - textbf{P}(B^{c}) + textbf{P}(A^{c})textbf{P}(B^{c}) geq 1 - textbf{P}(A^{c}) - textbf{P}(B^{c})
end{align*}










share|cite|improve this question











$endgroup$








  • 2




    $begingroup$
    $P(A)$ and $P(B)$ are numbers. How do you define $P(A)cap P(B)$?
    $endgroup$
    – ajotatxe
    Jan 13 at 20:58










  • $begingroup$
    Indeed as user ajotatxe remarks: already in the first line, the expression “$P(A)cap P(B)$” is meaningless since $P(cdot)$ is a number and $cap$ is a set operation.
    $endgroup$
    – LoveTooNap29
    Jan 13 at 21:04












  • $begingroup$
    You have assumed that $A$ and $B$ are independent events, but there is no need to.
    $endgroup$
    – user1337
    Jan 14 at 21:12
















0












$begingroup$


Let $A$ and $B$ be events defined on a sample space $Omega$. Prove that



$$textbf{P}(A cap B) geq 1 - textbf{P}(A^{c})- textbf{P}(B^{c}) $$



Answer:



begin{align*}
textbf{P}(Acap B) & = textbf{P}(A)textbf{P}(B) \
& = (1 - textbf{P}(A^{c}))(1 - textbf{P}(B^{c})) \
& = 1 - textbf{P}(A^{c}) - textbf{P}(B^{c}) + textbf{P}(A^{c})textbf{P}(B^{c}) geq 1 - textbf{P}(A^{c}) - textbf{P}(B^{c})
end{align*}










share|cite|improve this question











$endgroup$








  • 2




    $begingroup$
    $P(A)$ and $P(B)$ are numbers. How do you define $P(A)cap P(B)$?
    $endgroup$
    – ajotatxe
    Jan 13 at 20:58










  • $begingroup$
    Indeed as user ajotatxe remarks: already in the first line, the expression “$P(A)cap P(B)$” is meaningless since $P(cdot)$ is a number and $cap$ is a set operation.
    $endgroup$
    – LoveTooNap29
    Jan 13 at 21:04












  • $begingroup$
    You have assumed that $A$ and $B$ are independent events, but there is no need to.
    $endgroup$
    – user1337
    Jan 14 at 21:12














0












0








0





$begingroup$


Let $A$ and $B$ be events defined on a sample space $Omega$. Prove that



$$textbf{P}(A cap B) geq 1 - textbf{P}(A^{c})- textbf{P}(B^{c}) $$



Answer:



begin{align*}
textbf{P}(Acap B) & = textbf{P}(A)textbf{P}(B) \
& = (1 - textbf{P}(A^{c}))(1 - textbf{P}(B^{c})) \
& = 1 - textbf{P}(A^{c}) - textbf{P}(B^{c}) + textbf{P}(A^{c})textbf{P}(B^{c}) geq 1 - textbf{P}(A^{c}) - textbf{P}(B^{c})
end{align*}










share|cite|improve this question











$endgroup$




Let $A$ and $B$ be events defined on a sample space $Omega$. Prove that



$$textbf{P}(A cap B) geq 1 - textbf{P}(A^{c})- textbf{P}(B^{c}) $$



Answer:



begin{align*}
textbf{P}(Acap B) & = textbf{P}(A)textbf{P}(B) \
& = (1 - textbf{P}(A^{c}))(1 - textbf{P}(B^{c})) \
& = 1 - textbf{P}(A^{c}) - textbf{P}(B^{c}) + textbf{P}(A^{c})textbf{P}(B^{c}) geq 1 - textbf{P}(A^{c}) - textbf{P}(B^{c})
end{align*}







probability proof-verification






share|cite|improve this question















share|cite|improve this question













share|cite|improve this question




share|cite|improve this question








edited Jan 13 at 22:50









user1337

46110




46110










asked Jan 13 at 20:57









oamyoamy

1




1








  • 2




    $begingroup$
    $P(A)$ and $P(B)$ are numbers. How do you define $P(A)cap P(B)$?
    $endgroup$
    – ajotatxe
    Jan 13 at 20:58










  • $begingroup$
    Indeed as user ajotatxe remarks: already in the first line, the expression “$P(A)cap P(B)$” is meaningless since $P(cdot)$ is a number and $cap$ is a set operation.
    $endgroup$
    – LoveTooNap29
    Jan 13 at 21:04












  • $begingroup$
    You have assumed that $A$ and $B$ are independent events, but there is no need to.
    $endgroup$
    – user1337
    Jan 14 at 21:12














  • 2




    $begingroup$
    $P(A)$ and $P(B)$ are numbers. How do you define $P(A)cap P(B)$?
    $endgroup$
    – ajotatxe
    Jan 13 at 20:58










  • $begingroup$
    Indeed as user ajotatxe remarks: already in the first line, the expression “$P(A)cap P(B)$” is meaningless since $P(cdot)$ is a number and $cap$ is a set operation.
    $endgroup$
    – LoveTooNap29
    Jan 13 at 21:04












  • $begingroup$
    You have assumed that $A$ and $B$ are independent events, but there is no need to.
    $endgroup$
    – user1337
    Jan 14 at 21:12








2




2




$begingroup$
$P(A)$ and $P(B)$ are numbers. How do you define $P(A)cap P(B)$?
$endgroup$
– ajotatxe
Jan 13 at 20:58




$begingroup$
$P(A)$ and $P(B)$ are numbers. How do you define $P(A)cap P(B)$?
$endgroup$
– ajotatxe
Jan 13 at 20:58












$begingroup$
Indeed as user ajotatxe remarks: already in the first line, the expression “$P(A)cap P(B)$” is meaningless since $P(cdot)$ is a number and $cap$ is a set operation.
$endgroup$
– LoveTooNap29
Jan 13 at 21:04






$begingroup$
Indeed as user ajotatxe remarks: already in the first line, the expression “$P(A)cap P(B)$” is meaningless since $P(cdot)$ is a number and $cap$ is a set operation.
$endgroup$
– LoveTooNap29
Jan 13 at 21:04














$begingroup$
You have assumed that $A$ and $B$ are independent events, but there is no need to.
$endgroup$
– user1337
Jan 14 at 21:12




$begingroup$
You have assumed that $A$ and $B$ are independent events, but there is no need to.
$endgroup$
– user1337
Jan 14 at 21:12










3 Answers
3






active

oldest

votes


















1












$begingroup$

Notice that



begin{align*}
&textbf{P}(Acup B) leq 1 Leftrightarrow textbf{P}(A) + textbf{P}(B) - textbf{P}(Acap B) leq 1 Leftrightarrow\\
& 1 - textbf{P}(A^{c}) + 1 - textbf{P}(B^{c}) - textbf{P}(Acap B) leq 1 Leftrightarrow\\
& 1 - textbf{P}(A^{c}) - textbf{P}(B^{c}) leq textbf{P}(Acap B)
end{align*}



Where we have used the following properties
begin{cases}
textbf{P}(X) leq 1\\
textbf{P}(X) = 1 - textbf{P}(X^{c})\\
textbf{P}(Xcup Y) = textbf{P}(X) + textbf{P}(Y) - textbf{P}(Xcap Y)
end{cases}



Hope this helps.






share|cite|improve this answer









$endgroup$





















    1












    $begingroup$

    Note that $P(S^c)=1-P(S)$ and $P(Acup B)le 1$ therefore$$P(A)+P(B)-P(Acap B)le 1\P(A)+P(B)-1le P(Acap B)\P(A)+P(B)-1-1+1le P(Acap B)\-P(A^c)-P(B^c)+1le P(Acap B)$$and finally by rearranging the terms $$P(Acap B)ge 1-P(A^c)-P(B^c)$$with the equality if and only if $$Acup B=S$$






    share|cite|improve this answer









    $endgroup$





















      0












      $begingroup$

      You wrote:
      $$ textbf{P}(Acap B) = textbf{P}(A)textbf{P}(B) $$
      I claim this is wrong. Suppose that we are flipping a fair coin. Let $A$ be the event the
      coin comes up heads. Let $B$ be the event the coin comes up heads. This give us:
      begin{align*}
      textbf{P}(A) &= frac{1}{2} \
      textbf{P}(B) &= frac{1}{2} \
      textbf{P}(Acap B) &= frac{1}{2} \
      textbf{P}(A)textbf{P}(B) &= frac{1}{4} \
      end{align*}

      Hence their is a contradiction and your original statement is wrong.



      I hope this helps.



      Bob






      share|cite|improve this answer









      $endgroup$













        Your Answer





        StackExchange.ifUsing("editor", function () {
        return StackExchange.using("mathjaxEditing", function () {
        StackExchange.MarkdownEditor.creationCallbacks.add(function (editor, postfix) {
        StackExchange.mathjaxEditing.prepareWmdForMathJax(editor, postfix, [["$", "$"], ["\\(","\\)"]]);
        });
        });
        }, "mathjax-editing");

        StackExchange.ready(function() {
        var channelOptions = {
        tags: "".split(" "),
        id: "69"
        };
        initTagRenderer("".split(" "), "".split(" "), channelOptions);

        StackExchange.using("externalEditor", function() {
        // Have to fire editor after snippets, if snippets enabled
        if (StackExchange.settings.snippets.snippetsEnabled) {
        StackExchange.using("snippets", function() {
        createEditor();
        });
        }
        else {
        createEditor();
        }
        });

        function createEditor() {
        StackExchange.prepareEditor({
        heartbeatType: 'answer',
        autoActivateHeartbeat: false,
        convertImagesToLinks: true,
        noModals: true,
        showLowRepImageUploadWarning: true,
        reputationToPostImages: 10,
        bindNavPrevention: true,
        postfix: "",
        imageUploader: {
        brandingHtml: "Powered by u003ca class="icon-imgur-white" href="https://imgur.com/"u003eu003c/au003e",
        contentPolicyHtml: "User contributions licensed under u003ca href="https://creativecommons.org/licenses/by-sa/3.0/"u003ecc by-sa 3.0 with attribution requiredu003c/au003e u003ca href="https://stackoverflow.com/legal/content-policy"u003e(content policy)u003c/au003e",
        allowUrls: true
        },
        noCode: true, onDemand: true,
        discardSelector: ".discard-answer"
        ,immediatelyShowMarkdownHelp:true
        });


        }
        });














        draft saved

        draft discarded


















        StackExchange.ready(
        function () {
        StackExchange.openid.initPostLogin('.new-post-login', 'https%3a%2f%2fmath.stackexchange.com%2fquestions%2f3072509%2fprobability-proof-check-my-answer%23new-answer', 'question_page');
        }
        );

        Post as a guest















        Required, but never shown

























        3 Answers
        3






        active

        oldest

        votes








        3 Answers
        3






        active

        oldest

        votes









        active

        oldest

        votes






        active

        oldest

        votes









        1












        $begingroup$

        Notice that



        begin{align*}
        &textbf{P}(Acup B) leq 1 Leftrightarrow textbf{P}(A) + textbf{P}(B) - textbf{P}(Acap B) leq 1 Leftrightarrow\\
        & 1 - textbf{P}(A^{c}) + 1 - textbf{P}(B^{c}) - textbf{P}(Acap B) leq 1 Leftrightarrow\\
        & 1 - textbf{P}(A^{c}) - textbf{P}(B^{c}) leq textbf{P}(Acap B)
        end{align*}



        Where we have used the following properties
        begin{cases}
        textbf{P}(X) leq 1\\
        textbf{P}(X) = 1 - textbf{P}(X^{c})\\
        textbf{P}(Xcup Y) = textbf{P}(X) + textbf{P}(Y) - textbf{P}(Xcap Y)
        end{cases}



        Hope this helps.






        share|cite|improve this answer









        $endgroup$


















          1












          $begingroup$

          Notice that



          begin{align*}
          &textbf{P}(Acup B) leq 1 Leftrightarrow textbf{P}(A) + textbf{P}(B) - textbf{P}(Acap B) leq 1 Leftrightarrow\\
          & 1 - textbf{P}(A^{c}) + 1 - textbf{P}(B^{c}) - textbf{P}(Acap B) leq 1 Leftrightarrow\\
          & 1 - textbf{P}(A^{c}) - textbf{P}(B^{c}) leq textbf{P}(Acap B)
          end{align*}



          Where we have used the following properties
          begin{cases}
          textbf{P}(X) leq 1\\
          textbf{P}(X) = 1 - textbf{P}(X^{c})\\
          textbf{P}(Xcup Y) = textbf{P}(X) + textbf{P}(Y) - textbf{P}(Xcap Y)
          end{cases}



          Hope this helps.






          share|cite|improve this answer









          $endgroup$
















            1












            1








            1





            $begingroup$

            Notice that



            begin{align*}
            &textbf{P}(Acup B) leq 1 Leftrightarrow textbf{P}(A) + textbf{P}(B) - textbf{P}(Acap B) leq 1 Leftrightarrow\\
            & 1 - textbf{P}(A^{c}) + 1 - textbf{P}(B^{c}) - textbf{P}(Acap B) leq 1 Leftrightarrow\\
            & 1 - textbf{P}(A^{c}) - textbf{P}(B^{c}) leq textbf{P}(Acap B)
            end{align*}



            Where we have used the following properties
            begin{cases}
            textbf{P}(X) leq 1\\
            textbf{P}(X) = 1 - textbf{P}(X^{c})\\
            textbf{P}(Xcup Y) = textbf{P}(X) + textbf{P}(Y) - textbf{P}(Xcap Y)
            end{cases}



            Hope this helps.






            share|cite|improve this answer









            $endgroup$



            Notice that



            begin{align*}
            &textbf{P}(Acup B) leq 1 Leftrightarrow textbf{P}(A) + textbf{P}(B) - textbf{P}(Acap B) leq 1 Leftrightarrow\\
            & 1 - textbf{P}(A^{c}) + 1 - textbf{P}(B^{c}) - textbf{P}(Acap B) leq 1 Leftrightarrow\\
            & 1 - textbf{P}(A^{c}) - textbf{P}(B^{c}) leq textbf{P}(Acap B)
            end{align*}



            Where we have used the following properties
            begin{cases}
            textbf{P}(X) leq 1\\
            textbf{P}(X) = 1 - textbf{P}(X^{c})\\
            textbf{P}(Xcup Y) = textbf{P}(X) + textbf{P}(Y) - textbf{P}(Xcap Y)
            end{cases}



            Hope this helps.







            share|cite|improve this answer












            share|cite|improve this answer



            share|cite|improve this answer










            answered Jan 13 at 21:04









            user1337user1337

            46110




            46110























                1












                $begingroup$

                Note that $P(S^c)=1-P(S)$ and $P(Acup B)le 1$ therefore$$P(A)+P(B)-P(Acap B)le 1\P(A)+P(B)-1le P(Acap B)\P(A)+P(B)-1-1+1le P(Acap B)\-P(A^c)-P(B^c)+1le P(Acap B)$$and finally by rearranging the terms $$P(Acap B)ge 1-P(A^c)-P(B^c)$$with the equality if and only if $$Acup B=S$$






                share|cite|improve this answer









                $endgroup$


















                  1












                  $begingroup$

                  Note that $P(S^c)=1-P(S)$ and $P(Acup B)le 1$ therefore$$P(A)+P(B)-P(Acap B)le 1\P(A)+P(B)-1le P(Acap B)\P(A)+P(B)-1-1+1le P(Acap B)\-P(A^c)-P(B^c)+1le P(Acap B)$$and finally by rearranging the terms $$P(Acap B)ge 1-P(A^c)-P(B^c)$$with the equality if and only if $$Acup B=S$$






                  share|cite|improve this answer









                  $endgroup$
















                    1












                    1








                    1





                    $begingroup$

                    Note that $P(S^c)=1-P(S)$ and $P(Acup B)le 1$ therefore$$P(A)+P(B)-P(Acap B)le 1\P(A)+P(B)-1le P(Acap B)\P(A)+P(B)-1-1+1le P(Acap B)\-P(A^c)-P(B^c)+1le P(Acap B)$$and finally by rearranging the terms $$P(Acap B)ge 1-P(A^c)-P(B^c)$$with the equality if and only if $$Acup B=S$$






                    share|cite|improve this answer









                    $endgroup$



                    Note that $P(S^c)=1-P(S)$ and $P(Acup B)le 1$ therefore$$P(A)+P(B)-P(Acap B)le 1\P(A)+P(B)-1le P(Acap B)\P(A)+P(B)-1-1+1le P(Acap B)\-P(A^c)-P(B^c)+1le P(Acap B)$$and finally by rearranging the terms $$P(Acap B)ge 1-P(A^c)-P(B^c)$$with the equality if and only if $$Acup B=S$$







                    share|cite|improve this answer












                    share|cite|improve this answer



                    share|cite|improve this answer










                    answered Jan 13 at 21:41









                    Mostafa AyazMostafa Ayaz

                    17k31039




                    17k31039























                        0












                        $begingroup$

                        You wrote:
                        $$ textbf{P}(Acap B) = textbf{P}(A)textbf{P}(B) $$
                        I claim this is wrong. Suppose that we are flipping a fair coin. Let $A$ be the event the
                        coin comes up heads. Let $B$ be the event the coin comes up heads. This give us:
                        begin{align*}
                        textbf{P}(A) &= frac{1}{2} \
                        textbf{P}(B) &= frac{1}{2} \
                        textbf{P}(Acap B) &= frac{1}{2} \
                        textbf{P}(A)textbf{P}(B) &= frac{1}{4} \
                        end{align*}

                        Hence their is a contradiction and your original statement is wrong.



                        I hope this helps.



                        Bob






                        share|cite|improve this answer









                        $endgroup$


















                          0












                          $begingroup$

                          You wrote:
                          $$ textbf{P}(Acap B) = textbf{P}(A)textbf{P}(B) $$
                          I claim this is wrong. Suppose that we are flipping a fair coin. Let $A$ be the event the
                          coin comes up heads. Let $B$ be the event the coin comes up heads. This give us:
                          begin{align*}
                          textbf{P}(A) &= frac{1}{2} \
                          textbf{P}(B) &= frac{1}{2} \
                          textbf{P}(Acap B) &= frac{1}{2} \
                          textbf{P}(A)textbf{P}(B) &= frac{1}{4} \
                          end{align*}

                          Hence their is a contradiction and your original statement is wrong.



                          I hope this helps.



                          Bob






                          share|cite|improve this answer









                          $endgroup$
















                            0












                            0








                            0





                            $begingroup$

                            You wrote:
                            $$ textbf{P}(Acap B) = textbf{P}(A)textbf{P}(B) $$
                            I claim this is wrong. Suppose that we are flipping a fair coin. Let $A$ be the event the
                            coin comes up heads. Let $B$ be the event the coin comes up heads. This give us:
                            begin{align*}
                            textbf{P}(A) &= frac{1}{2} \
                            textbf{P}(B) &= frac{1}{2} \
                            textbf{P}(Acap B) &= frac{1}{2} \
                            textbf{P}(A)textbf{P}(B) &= frac{1}{4} \
                            end{align*}

                            Hence their is a contradiction and your original statement is wrong.



                            I hope this helps.



                            Bob






                            share|cite|improve this answer









                            $endgroup$



                            You wrote:
                            $$ textbf{P}(Acap B) = textbf{P}(A)textbf{P}(B) $$
                            I claim this is wrong. Suppose that we are flipping a fair coin. Let $A$ be the event the
                            coin comes up heads. Let $B$ be the event the coin comes up heads. This give us:
                            begin{align*}
                            textbf{P}(A) &= frac{1}{2} \
                            textbf{P}(B) &= frac{1}{2} \
                            textbf{P}(Acap B) &= frac{1}{2} \
                            textbf{P}(A)textbf{P}(B) &= frac{1}{4} \
                            end{align*}

                            Hence their is a contradiction and your original statement is wrong.



                            I hope this helps.



                            Bob







                            share|cite|improve this answer












                            share|cite|improve this answer



                            share|cite|improve this answer










                            answered Jan 13 at 23:05









                            BobBob

                            938515




                            938515






























                                draft saved

                                draft discarded




















































                                Thanks for contributing an answer to Mathematics Stack Exchange!


                                • Please be sure to answer the question. Provide details and share your research!

                                But avoid



                                • Asking for help, clarification, or responding to other answers.

                                • Making statements based on opinion; back them up with references or personal experience.


                                Use MathJax to format equations. MathJax reference.


                                To learn more, see our tips on writing great answers.




                                draft saved


                                draft discarded














                                StackExchange.ready(
                                function () {
                                StackExchange.openid.initPostLogin('.new-post-login', 'https%3a%2f%2fmath.stackexchange.com%2fquestions%2f3072509%2fprobability-proof-check-my-answer%23new-answer', 'question_page');
                                }
                                );

                                Post as a guest















                                Required, but never shown





















































                                Required, but never shown














                                Required, but never shown












                                Required, but never shown







                                Required, but never shown

































                                Required, but never shown














                                Required, but never shown












                                Required, but never shown







                                Required, but never shown







                                Popular posts from this blog

                                Human spaceflight

                                Can not write log (Is /dev/pts mounted?) - openpty in Ubuntu-on-Windows?

                                File:DeusFollowingSea.jpg